[Rozgrzewka OM][MIX][Nierówności] Nierówności

Zadania z kółek matematycznych lub obozów przygotowujących do OM. Problemy z minionych olimpiad i konkursów matematycznych.
Regulamin forum
Wszystkie tematy znajdujące się w tym dziale powinny być tagowane tj. posiadać przedrostek postaci [Nierówności], [Planimetria], itp.. Temat może posiadać wiele różnych tagów. Nazwa tematu nie może składać się z samych tagów.
Thingoln
Użytkownik
Użytkownik
Posty: 133
Rejestracja: 27 lip 2019, o 22:19
Płeć: Mężczyzna
Lokalizacja: Warszawa
Podziękował: 52 razy
Pomógł: 15 razy

Re: [Rozgrzewka OM][MIX][Nierówności] Nierówności

Post autor: Thingoln »

Niech \(\displaystyle{ a_1, a_2, \dots, a_6}\) będą liczbami dodatnimi, dla których jest spełnione \(\displaystyle{ 1 = a_1 a_2 \dots a_6}\). Udowodnić nierówność \(\displaystyle{ \frac{1}{a_1(1+a_2)} + \frac{1}{a_2(1+a_3)} + \dots + \frac{1}{a_6(1+a_1)} \ge 3}\).
Awatar użytkownika
Premislav
Użytkownik
Użytkownik
Posty: 15685
Rejestracja: 17 sie 2012, o 13:12
Płeć: Mężczyzna
Lokalizacja: Warszawa
Podziękował: 195 razy
Pomógł: 5219 razy

Re: [Rozgrzewka OM][MIX][Nierówności] Nierówności

Post autor: Premislav »

Ukryta treść:    
Thingoln
Użytkownik
Użytkownik
Posty: 133
Rejestracja: 27 lip 2019, o 22:19
Płeć: Mężczyzna
Lokalizacja: Warszawa
Podziękował: 52 razy
Pomógł: 15 razy

Re: [Rozgrzewka OM][MIX][Nierówności] Nierówności

Post autor: Thingoln »

Wygląda super. :D Nierówności Shapiro nie znałem, ale wydaje się warta zapamiętania. Możesz wrzucać kolejne.
Awatar użytkownika
Premislav
Użytkownik
Użytkownik
Posty: 15685
Rejestracja: 17 sie 2012, o 13:12
Płeć: Mężczyzna
Lokalizacja: Warszawa
Podziękował: 195 razy
Pomógł: 5219 razy

Re: [Rozgrzewka OM][MIX][Nierówności] Nierówności

Post autor: Premislav »

Nowe:
Dla ustalonego \(\displaystyle{ n\in\NN, \ n\ge 2}\) proszę znaleźć największą stałą \(\displaystyle{ C(n)}\), dla której nierówność \(\displaystyle{ \sum_{i=1}^{n}x_{i}\ge C(n)\sum_{1\le i<j\le n}^{}\left(2x_{i}x_{j}+\sqrt{x_{i}x_{j}}\right)}\)
zachodzi dla dowolnych \(\displaystyle{ x_{i}\in(0,1)}\) spełniających warunek \(\displaystyle{ (1-x_{i})(1-x_{j})\ge \frac{1}{4}, \ 1\le i<j \le n}\)
Tmkk
Użytkownik
Użytkownik
Posty: 1718
Rejestracja: 15 wrz 2010, o 15:36
Płeć: Mężczyzna
Lokalizacja: Ostrołęka
Podziękował: 59 razy
Pomógł: 501 razy

Re: [Rozgrzewka OM][MIX][Nierówności] Nierówności

Post autor: Tmkk »

Ukryta treść:    
Awatar użytkownika
Premislav
Użytkownik
Użytkownik
Posty: 15685
Rejestracja: 17 sie 2012, o 13:12
Płeć: Mężczyzna
Lokalizacja: Warszawa
Podziękował: 195 razy
Pomógł: 5219 razy

Re: [Rozgrzewka OM][MIX][Nierówności] Nierówności

Post autor: Premislav »

Bardzo zgrabnie, możesz wrzucać następne zadanko,
Tmkk
Użytkownik
Użytkownik
Posty: 1718
Rejestracja: 15 wrz 2010, o 15:36
Płeć: Mężczyzna
Lokalizacja: Ostrołęka
Podziękował: 59 razy
Pomógł: 501 razy

Re: [Rozgrzewka OM][MIX][Nierówności] Nierówności

Post autor: Tmkk »

Dzięki : ) To może coś takiego. Lekka modyfikacja czegoś, co już pewnie widzieliście, mam nadzieję, że nadal prawdziwa...

Pokazać, że dla boków trójkąta \(\displaystyle{ a,b,c}\) zachodzi

\(\displaystyle{ \frac{a}{2a+c-b} + \frac{b}{2b+a-c} + \frac{c}{2c+b-a} \ge \frac{3}{2}}\).
Thingoln
Użytkownik
Użytkownik
Posty: 133
Rejestracja: 27 lip 2019, o 22:19
Płeć: Mężczyzna
Lokalizacja: Warszawa
Podziękował: 52 razy
Pomógł: 15 razy

Re: [Rozgrzewka OM][MIX][Nierówności] Nierówności

Post autor: Thingoln »

Ukryta treść:    
Awatar użytkownika
Premislav
Użytkownik
Użytkownik
Posty: 15685
Rejestracja: 17 sie 2012, o 13:12
Płeć: Mężczyzna
Lokalizacja: Warszawa
Podziękował: 195 razy
Pomógł: 5219 razy

Re: [Rozgrzewka OM][MIX][Nierówności] Nierówności

Post autor: Premislav »

Ukryta treść:    
Tmkk
Użytkownik
Użytkownik
Posty: 1718
Rejestracja: 15 wrz 2010, o 15:36
Płeć: Mężczyzna
Lokalizacja: Ostrołęka
Podziękował: 59 razy
Pomógł: 501 razy

Re: [Rozgrzewka OM][MIX][Nierówności] Nierówności

Post autor: Tmkk »

Tak, dokładnie o to chodziło.
Awatar użytkownika
Premislav
Użytkownik
Użytkownik
Posty: 15685
Rejestracja: 17 sie 2012, o 13:12
Płeć: Mężczyzna
Lokalizacja: Warszawa
Podziękował: 195 razy
Pomógł: 5219 razy

Re: [Rozgrzewka OM][MIX][Nierówności] Nierówności

Post autor: Premislav »

Nowe zadanie: dla rzeczywistych dodatnich \(\displaystyle{ a,b,c}\) spełniających warunek
\(\displaystyle{ 3\max\left\{a^{2}, b^{2}, c^{2}\right\}\le 2\left(a^{2}+b^{2}+c^{2}\right)}\) proszę wykazać nierówność
\(\displaystyle{ \frac{a}{\sqrt{2b^{2}+2c^{2}-a^{2}}}+\frac{b}{\sqrt{2c^{2}+2a^{2}-b^{2}}}+\frac{c}{\sqrt{2a^{2}+2b^{2}-c^{2}}}\ge \sqrt{3}}\)
bosa_Nike
Użytkownik
Użytkownik
Posty: 1660
Rejestracja: 16 cze 2006, o 15:40
Płeć: Kobieta
Podziękował: 70 razy
Pomógł: 445 razy

Re: [Rozgrzewka OM][MIX][Nierówności] Nierówności

Post autor: bosa_Nike »

Korzystając ze, z pewnością chwilowego, zwolnienia tempa, pozwolę sobie wrzucić alternatywne rozwiązanie poprzedniego problemu. Nie jest to w żadnym razie rozwiązanie prostsze ani szybsze w tym przypadku, ale chcę sprzedać pewien trick, który tu akurat działa, a który w mojej ocenie warto mieć w arsenale na inne zadania.
Poprzednie:    
Tmkk
Użytkownik
Użytkownik
Posty: 1718
Rejestracja: 15 wrz 2010, o 15:36
Płeć: Mężczyzna
Lokalizacja: Ostrołęka
Podziękował: 59 razy
Pomógł: 501 razy

Re: [Rozgrzewka OM][MIX][Nierówności] Nierówności

Post autor: Tmkk »

Co to obecnego zadania:
Ukryta treść:    
Awatar użytkownika
Premislav
Użytkownik
Użytkownik
Posty: 15685
Rejestracja: 17 sie 2012, o 13:12
Płeć: Mężczyzna
Lokalizacja: Warszawa
Podziękował: 195 razy
Pomógł: 5219 razy

Re: [Rozgrzewka OM][MIX][Nierówności] Nierówności

Post autor: Premislav »

No i fajnie. Hung w I tomie Secrets in Inequalities proponuje dowód z Höldera, ale Twoje rozwiązanie jest prostsze.
Tmkk
Użytkownik
Użytkownik
Posty: 1718
Rejestracja: 15 wrz 2010, o 15:36
Płeć: Mężczyzna
Lokalizacja: Ostrołęka
Podziękował: 59 razy
Pomógł: 501 razy

Re: [Rozgrzewka OM][MIX][Nierówności] Nierówności

Post autor: Tmkk »

Oki, to kolejne zadanie:

Niech \(\displaystyle{ a,b,c}\) będą liczbami rzeczywistymi dodatnimi oraz \(\displaystyle{ a+b+c=1}\). Pokazać, że zachodzi

\(\displaystyle{ \left(\frac{1}{a}+\frac{1}{bc}\right)\left(\frac{1}{b}+\frac{1}{ca}\right)\left(\frac{1}{c}+\frac{1}{ab}\right) \ge 1728}\).
ODPOWIEDZ